next up previous
Next: About this document ...

Sequence Convergence and Divergence
Examples

Today in class, I proved that the sequence $ \left\{
\int_{1+\frac{1}{n}}^{2-\frac{1}{n}} \frac{1}{x} \: dx \right\} ^{\infty}_{n=k}$ converges to a limit of $\ln{2}$.

There are two steps in working such a problem:
1. Find the limit, L (i.e. using calculus techniques).
2. Prove that the limit exists, and is equal to L.

Step 1: Find the limit.


\begin{eqnarray*}
\lim_{n \to \infty} \int_{1+\frac{1}{n}}^{2-\frac{1}{n}} \frac...
... \infty}\frac{2n - 1}{n + 1} \Biggr) } \\ [.1 in]
& = & \ln{2}.
\end{eqnarray*}


Now that we have computed what the limit is (or rather, at this point we ought to say ``what the limit would be, if it exists''), we have to prove that the limit exists, and that it is in fact, $\ln{2}$.

Step 2: Prove the limit is $\ln{2}.$

Let us recall the definition of convergence: a sequence $\{a_n\}_{n=k}^{\infty}$ converges to a limit, L, if, for every $\epsilon > 0$, there exists a real number, M (a function of $\epsilon$), such that if n > M, then $\vert a_n - L\vert < \epsilon$.

What this means is that in order to prove our sequence converges, we need to demonstrate that when someone hands us an $\epsilon$ (think: really small number), we can give them back an M (think: really large number, usually expressed in terms of $\epsilon$, and NOT involving n), so that as long as we are looking at terms of the sequence past the Mth (i.e. as long as n > M), then we can guarantee that $\vert a_n - L\vert < \epsilon$.

So how do we go about finding such an M? Well, we assume that what we want to be true is true, for the time being. What we want is for


\begin{displaymath}\Biggl\vert \int_{1+\frac{1}{n}}^{2-\frac{1}{n}} \frac{1}{x} \: dx - \ln{2}
\Biggr\vert < \epsilon,\end{displaymath}

when n is greater than some M (which we get to pick). How do we find such an M?

Well, the inequality above tells us exactly what restriction we need to place on n to ensure we are no less than $\epsilon$ away from our limit. We just need to solve the inequality for n so that we can see it explicitly.

Having already determined that


\begin{displaymath}\lim_{n \to \infty} \int_{1+\frac{1}{n}}^{2-\frac{1}{n}} \frac{1}{x}
\: dx = \ln{\Biggl( \frac{2n - 1}{n + 1} \Biggr) },\end{displaymath}

we begin working backwards to find n:


\begin{displaymath}\Biggl\vert \ln{\Biggl( \frac{2n - 1}{n + 1} \Biggr) } - \ln{2} \Biggr\vert
< \epsilon\end{displaymath}

Now, as I said in class, the stuff inside the absolute value bars is actually a negative number (since $\frac{2n-1}{n+1} < 2$), so we switch the order of subtraction to make it positive...

\begin{displaymath}\Biggl\vert \ln{2} - \ln{\Biggl( \frac{2n - 1}{n + 1} \Biggr) } \Biggr\vert
< \epsilon\end{displaymath}

Now, use the logarithm subtraction formula ( $\ln{a} - \ln{b} =
\ln{\frac{a}{b}}$), noting that absolute value bars are no longer needed:

\begin{displaymath}\ln{\Biggl( \frac{2n+2}{2n-1} \Biggr)} < \epsilon\end{displaymath}

Next, exponentiate both sides

\begin{displaymath}\frac{2n+2}{2n-1} < e^{\epsilon}\end{displaymath}

and perform long division on the left-hand side:

\begin{displaymath}1 + \frac{3}{2n-1} < e^{\epsilon}\end{displaymath}

and finally solve for n:

\begin{displaymath}\frac{3}{2n-1} < e^{\epsilon} - 1\end{displaymath}


\begin{displaymath}\frac{3}{e^{\epsilon} - 1} < 2n - 1\end{displaymath}


\begin{displaymath}n > \frac{3}{2(e^{\epsilon} - 1)} + \frac{1}{2}\end{displaymath}

What does this last statement say? It says that if we assume for the moment that we have some value of n such that


\begin{displaymath}\Biggl\vert \int_{1+\frac{1}{n}}^{2-\frac{1}{n}} \frac{1}{x} \: dx - \ln{2}
\Biggr\vert < \epsilon,\end{displaymath}

then it logically follows that


\begin{displaymath}n > \frac{3}{2(e^{\epsilon} - 1)} + \frac{1}{2}.\end{displaymath}

Hence, if given some $\epsilon > 0$, we set

\begin{displaymath}M = \frac{3}{2(e^{\epsilon} - 1)} + \frac{1}{2},\end{displaymath}

then whenever n > M, we will have

\begin{displaymath}\Biggl\vert \int_{1+\frac{1}{n}}^{2-\frac{1}{n}} \frac{1}{x} \: dx - \ln{2}
\Biggr\vert < \epsilon.\end{displaymath}

This is exactly what we were trying to prove.

Several people have queried me about what would happen if we tried to prove that the limit of this sequence was $\ln{3}$. How would you know you went wrong? Well, let's try to repeat the calculations, this time using $\ln{3}$.


\begin{displaymath}\Biggl\vert \ln{\Biggl( \frac{2n - 1}{n + 1} \Biggr) } - \ln{3} \Biggr\vert
< \epsilon\end{displaymath}

Again, since $\frac{2n-1}{n+1} < 3$, the stuff inside the absolute value bars is negative, so we reverse the order...

\begin{displaymath}\Biggl\vert \ln{3} - \ln{\Biggl( \frac{2n - 1}{n + 1} \Biggr) } \Biggr\vert
< \epsilon\end{displaymath}

Now, use the logarithm subtraction formula ( $\ln{a} - \ln{b} =
\ln{\frac{a}{b}}$), noting that absolute value bars are no longer needed:

\begin{displaymath}\ln{\Biggl( \frac{3n+3}{2n-1} \Biggr)} < \epsilon\end{displaymath}

Next, exponentiate both sides

\begin{displaymath}\frac{3n+3}{2n-1} < e^{\epsilon}\end{displaymath}

and perform long division on the left-hand side:

\begin{displaymath}\frac{3}{2} + \frac{9}{2(2n-1)} < e^{\epsilon}\end{displaymath}

and finally solve for n:

\begin{displaymath}\frac{9}{4n-2} < e^{\epsilon} - \frac{3}{2}\end{displaymath}


\begin{displaymath}\frac{9}{e^{\epsilon} - \frac{3}{2}} < 4n - 2\end{displaymath}


\begin{displaymath}n > \frac{9}{4e^{\epsilon} - 6} + \frac{1}{2}\end{displaymath}

Let's call the right-hand side of this inequality M'.

At first glance, this may look just as legitimate as the last example, but it isn't. Here's a guideline that should be pretty easy to use. As we said before, someone is handing you an $\epsilon$, and you have to find an M that's going to guarantee the sequence terms are closer than $\epsilon$ to the limit value. It should make sense, then, that the smaller the value of $\epsilon$, the higher you need to make M in order to get close enough to the limit value. In fact, as $\epsilon \rightarrow 0$, you should expect that $M \rightarrow
\infty$. Let's try this for our original M:


\begin{displaymath}\lim_{\epsilon \to 0} \frac{3}{2(e^{\epsilon} - 1)} = \infty\end{displaymath}

The denominator goes to zero (from the positive side), hence we get $\infty$ for the limit.

Now, let's try it for the value M' we computed using $\ln{3}$ for the limit:


\begin{displaymath}\lim_{\epsilon \to 0} \frac{9}{4e^{\epsilon} - 6} + \frac{1}{2} =
\frac{9}{4(1) - 6} + \frac{1}{2} = -4.\end{displaymath}

Hmmm...that doesn't seem to make much sense. If this were true, then if $\epsilon$ is really small, we should be able to pick any n value greater than -4 and find we are within $\epsilon$ of our limit value, $\ln{3}$. Suppose $\epsilon = .001$. Then


\begin{displaymath}M' = \frac{9}{4e^{.001} - 6} \approx -4.509\end{displaymath}

So we should be able to pick any n > -4.509, and have our nth term (and beyond) be within .001 of $\ln{3}$. So let's try it for n = 2.


\begin{displaymath}\Biggl\vert \ln{\Biggl( \frac{2(2) - 1}{(2) + 1} \Biggr) } - ...
... \Biggr\vert
= \vert\ln{1} - \ln{3}\vert = \ln{3} \nless .001\end{displaymath}

If you've lasted this long, you may be wondering ``but so what? You just chose a bad value for $\epsilon$ and n.'' But if we got back and look at the definition of covergence, the M we choose must work for any choice of $\epsilon$, and for any n > M. Since it doesn't, we must conclude that our original assumption, that the limit of the sequence was $\ln{3}$, must be wrong.




next up previous
Next: About this document ...
Math 23 Winter 2000
2000-01-13